Ist es wirklich richtig zu sagen, dass die Identität der Gemeinde eine Folge der Eichinvarianz ist?

Viele (wenn nicht alle) der Materialien, die ich gelesen habe, behaupten, dass die Identität von Ward eine Folge der Eichinvarianz der Theorie ist, während ihre Ableitungen tatsächlich nur die aktuelle Erhaltung nutzen μ J. μ = 0 μ J. μ = 0 (was nur einer globalen Phasensymmetrie entspricht). Ich bin mir der Tatsache bewusst, dass ein Messfeld an einen konservierten Strom gekoppelt werden muss, um die Messinvarianz aufrechtzuerhalten, aber ein Nicht-Messfeld kann auch an einen konservierten Strom und in diesem Fall an die Ward-Identität gekoppelt werden (muss es aber nicht) sollte noch halten. Halten Sie es also für irreführend, wenn nicht sogar falsch, zu behaupten, die Identität der Gemeinde sei eine Folge der Eichinvarianz?

Vielleicht könnten Sie ein Beispiel zeigen, wo der Strom J. J. ist genau erhalten und koppelt an ein Feld EIN EIN über J. J. , aber die resultierende Theorie hat keine Eichensymmetrie?
Und wo auch die Identität der Gemeinde beachtet wird.
@BebopButUnsteady: Wie wäre es mit einem massiven Vektorfeld? EIN μ EIN μ gekoppelt mit etwas konserviertem Strom J. μ J. μ ? Der Massenbegriff m 2 EIN μ EIN μ m 2 EIN μ EIN μ verletzt die Eichensymmetrie, aber die Identität von Ward ist immer noch wahr, weil J. μ J. μ ist erhalten.

Antworten (3)

Diese Antwort stimmt teilweise nicht mit der von Motl überein. Der entscheidende Punkt ist, den Unterschied zwischen dem abelschen und dem nicht-abelschen Fall zu berücksichtigen. Ich stimme Motls Antwort im nicht-abelschen Fall voll und ganz zu - wo diese Identitäten normalerweise eher als Slavnov-Taylor als als als Ward bezeichnet werden, so dass ich mich auf den abelschen Fall beziehen werde.

Zunächst ein paar Worte zur Terminologie: Ward-Identitäten sind das Quantengegenstück zum (ersten und zweiten) Noether-Theorem in der klassischen Physik. Sie gelten sowohl für globale als auch für Eichsymmetrien. Der Begriff ist jedoch häufig dem vorbehalten U. ( 1 ) U. ( 1 ) Eichsymmetrie in QED. Im Fall von Eichensymmetrien ergeben Ward-Identitäten reale Identitäten, wie z k μ M. μ = 0 k μ M. μ = 0 , wo M. μ M. μ wird definiert durch M = ϵ μ M. μ M. = ϵ μ M. μ In QED heißt es, dass die Polarisationen des Photons parallel zur Ausbreitung des Photons nicht zur Streuamplitude beitragen. Bei globalen Symmetrien spiegeln die Ward-Identitäten jedoch die Eigenschaften der Theorie wider. Zum Beispiel ist die S-Matrix einer Lorentz-Invarianten-Theorie auch Lorentz-Invarianten oder die Anzahl der Teilchen minus Antiteilchen im Anfangszustand ist dieselbe wie im Endzustand in einer Theorie mit einem globalen (unabhängig vom Punkt in der Raumzeit) ) U. ( 1 ) U. ( 1 ) Phaseninvarianz.

Lassen Sie uns den Fall eines massiven Vektorfeldes untersuchen, das minimal an einen konservierten Strom gekoppelt ist:

L = - 1 4 F. 2 + a 2 2 EIN 2 + i Ψ ¯ /. D. Ψ - m 2 2 Ψ ¯ Ψ = - 1 4 F. 2 + a 2 2 EIN 2 + i Ψ ¯ /. Ψ - m 2 2 Ψ ¯ Ψ - e EIN μ j μ L. = - - 1 4 F. 2 + ein 2 2 EIN 2 + ich Ψ ¯ D. Ψ - - m 2 2 Ψ ¯ Ψ = - - 1 4 F. 2 + ein 2 2 EIN 2 + ich Ψ ¯ Ψ - - m 2 2 Ψ ¯ Ψ - - e EIN μ j μ

Beachten Sie, dass diese Theorie eine globale Phaseninvarianz aufweist Ψ e - i θ Ψ Ψ e - - ich θ Ψ mit einem Noetherstrom

j μ = Ψ ¯ γ μ Ψ j μ = Ψ ¯ γ μ Ψ

so dass (klassisch) μ j μ = 0 μ j μ = 0 . Abgesehen von dieser Symmetrie ist bekannt, dass der obige Lagrange einer Theorie entspricht: i) die keinen expliziten Massenterm für das Vektorfeld hat. ii) das ein Skalarfeld (ein Higgs-ähnliches Feld) mit einem von Null verschiedenen Vakuumerwartungswert enthält, das a spontan bricht U. ( 1 ) U. ( 1 ) Eichsymmetrie (diese Symmetrie ist nicht die gemessene U. ( 1 ) U. ( 1 ) globale Symmetrie zuvor erwähnt). Die Äquivalenz liegt an der Grenze, an der der Vakuumerwartungswert gegen unendlich geht und die Kopplung zwischen dem Vektorfeld und dem Higgs-ähnlichen Skalar gegen Null geht. Da man diese letzte Grenze nehmen muss, kann die Ladung nicht quantisiert werden und daher die U. ( 1 ) U. ( 1 ) Die Eichsymmetrie muss topologisch der Addition reeller Zahlen entsprechen und nicht der Multiplikation komplexer Zahlen mit dem Einheitsmodul (einem Umfang). Der Unterschied zwischen beiden Gruppen ist nur topologisch (bedeutet dies dann, dass der Unterschied im Folgenden irrelevant ist?). Dieser Mechanismus ist auf Stückelberg zurückzuführen, und ich werde ihn am Ende dieser Antwort zusammenfassen.

In einem Prozess, in dem sich im Anfangs- oder Endzustand ein massives Vektorpartikel befindet, ergibt die LSZ-Reduktionsformel:

⟨I | f ϵ μ d 4 x e - i k x ( η μ ν ( 2 - a 2 ) - μ ν ) . . . ⟨0 | T. EIN ν ( x ) . . . | 0⟩ ich | f ϵ μ d 4 x e - - ich k x ( η μ ν ( 2 - - ein 2 ) - - μ ν ) . . . 0 | T. EIN ν ( x ) . . . | 0

Aus dem obigen Lagrange können die folgenden klassischen Bewegungsgleichungen erhalten werden

( η μ ν ( 2 - a 2 ) - μ ν ) A. ν = e j μ ( η μ ν ( 2 - - ein 2 ) - - μ ν ) EIN ν = e j μ

Dann quantenmäßig

( η μ ν ( 2 - a 2 ) - μ ν ) ⟨0 | T. EIN ν ( x ) . . . | 0⟩ = e ⟨0 | T. j μ ( x ) . . . | 0⟩ + Kontaktbedingungen, die nicht zur S-Matrix beitragen ( η μ ν ( 2 - - ein 2 ) - - μ ν ) 0 | T. EIN ν ( x ) . . . | 0 = e 0 | T. j μ ( x ) . . . | 0 + Kontaktbedingungen, die nicht zur S-Matrix beitragen

Und deshalb

⟨I | f ϵ μ d 4 x e - i k x . . . ⟨0 | T. j μ ( x ) . . . | 0⟩ + Kontaktbedingungen, die nicht ∼ ∼ beitragen μ M. μ ich | f ϵ μ d 4 x e - - ich k x . . . 0 | T. j μ ( x ) . . . | 0 + Kontaktbedingungen, die nicht dazu beitragen ϵ μ M. μ

Wenn man ersetzt ϵ μ ϵ μ mit k μ k μ , Man erhält

k μ M. μ k μ d 4 x e - i k x . . . ⟨0 | T. j μ ( x ) . . . | 0⟩ k μ M. μ k μ d 4 x e - - ich k x . . . 0 | T. j μ ( x ) . . . | 0

Gebrauch machen von k μ μ e - i k x k μ μ , e - - ich k x Wenn man sich nach Teilen integriert und den Oberflächenterm reitet (die ebene Welle ist eine Idealisierung, was man tatsächlich hat, ist ein Wellenpaket, das im räumlichen Unendlichen auf Null geht), bekommt man

k μ M. μ ∼ ∼ d 4 x e - i k x . . . μ ⟨0 | T. j μ ( x ) . . . | 0⟩ k μ M. μ d 4 x e - - ich k x . . . μ 0 | T. j μ ( x ) . . . | 0

Man kann jetzt die Ward-Identität für das Globale verwenden Ψ e - i θ Ψ Ψ e - - ich θ Ψ Symmetrie (klassisch μ j μ = 0 μ j μ = 0 über Lösungen der Sache, Ψ Ψ Bewegungsgleichungen)

μ ⟨0 | T. j μ ( x ) . . . | 0⟩ = Kontaktbedingungen, die nicht zur S-Matrix beitragen μ 0 | T. j μ ( x ) . . . | 0 = Kontaktbedingungen, die nicht zur S-Matrix beitragen

Und daher

k μ M. μ = 0 k μ M. μ = 0

das gleiche wie im masselosen Fall.

Beachten Sie, dass es bei dieser Ableitung entscheidend war, dass der explizite Massenterm für das Vektorfeld das Globale nicht bricht U. ( 1 ) U. ( 1 ) Symmetrie. Dies hängt auch damit zusammen, dass der explizite Massenterm für das Vektorfeld durch einen Higgs-ähnlichen Mechanismus erhalten werden kann, der mit einem versteckten verbunden ist (das Higgs-ähnliche Feld entkoppelt sich vom Rest der Theorie). U. ( 1 ) U. ( 1 ) Eichsymmetrie.

Eine sorgfältigere Berechnung sollte Gegensätze in die Interaktionstheorie einbeziehen, ich denke jedoch, dass dies das gleiche ist wie im masselosen Fall. Wir können uns die Felder und Parameter in dieser Antwort als bloße Felder und Parameter vorstellen.

Stueckelberg-Mechanismus

Betrachten Sie den folgenden Lagrange

L = - 1 4 F. 2 + | d ϕ | 2 + μ 2 | ϕ | 2 - λ ( ϕ ϕ ) 2 L. = - - 1 4 F. 2 + | d ϕ | 2 + μ 2 | ϕ | 2 - - λ ( ϕ ϕ ) 2

wo d = - ich g B. d = - - ich G B. und F. F. ist die Feldstärke (Faradayscher Tensor) für B. B. . Dieser Lagrange ist unter der Eichentransformation invariant

B B + ( 1 / g ) α ( x ) B. B. + ( 1 /. G ) α ( x )
ϕ e i α ( x ) ϕ ϕ e ich α ( x ) ϕ

Nehmen wir eine polare Parametrisierung für das Skalarfeld ϕ ϕ :: ϕ 1 2 ρ e ich χ ϕ 1 2 ρ e ich χ also

L = - 1 4 F. 2 + 1 2 ρ 2 ( μ χ - g B. μ ) 2 + 1 2 ( ρ ) 2 + μ 2 2 ρ 2 - λ 4 ρ 4 L. = - - 1 4 F. 2 + 1 2 ρ 2 ( μ χ - - G B. μ ) 2 + 1 2 ( ρ ) 2 + μ 2 2 ρ 2 - - λ 4 ρ 4

Wir können nun die folgende Feldneudefinition vornehmen A B - ( 1 / g ) χ EIN B. - - ( 1 /. G ) χ und das zu bemerken F. μ ν = μ B. ν - ν B. μ = μ EIN ν - ν EIN μ F. μ ν = μ B. ν - - ν B. μ = μ EIN ν - - ν EIN μ ist auch die Feldstärke für EIN EIN

L = - 1 4 F. 2 + g 2 2 ρ 2 EIN 2 + 1 2 ( ρ ) 2 + μ 2 2 ρ 2 - λ 4 ρ 4 L. = - - 1 4 F. 2 + G 2 2 ρ 2 EIN 2 + 1 2 ( ρ ) 2 + μ 2 2 ρ 2 - - λ 4 ρ 4

Wenn ρ ρ hat einen von Null verschiedenen Vakuumerwartungswert ⟨0 | ρ | 0⟩ = v = μ 2 λ - - - - 0 | ρ | 0 = v = μ 2 λ ist es dann bequem zu schreiben ρ ( x ) = v + ω ( x ) ρ ( x ) = v + ω ( x ) . Somit

L = - 1 4 F. 2 + a 2 2 EIN 2 + g 2 v ω EIN 2 + g 2 2 ω 2 EIN 2 + 1 2 ( ω ) 2 - μ 2 2 ω 2 - λ v ω 3 - λ 4 ω 4 + v 4 λ 2 4 L. = - - 1 4 F. 2 + ein 2 2 EIN 2 + G 2 v ω EIN 2 + G 2 2 ω 2 EIN 2 + 1 2 ( ω ) 2 - - μ 2 2 ω 2 - - λ v ω 3 - - λ 4 ω 4 + v 4 λ 2 4

wo a g × v ein G × v . Wenn wir jetzt das Limit nehmen G 0 G 0 , v v , das Produkt behalten, ein ein konstant bekommen wir

L = - 1 4 F. 2 + a 2 2 EIN 2 + 1 2 ( ω ) 2 - μ 2 2 ω 2 - λ v ω 3 - λ 4 ω 4 + v 4 λ 2 4 L. = - - 1 4 F. 2 + ein 2 2 EIN 2 + 1 2 ( ω ) 2 - - μ 2 2 ω 2 - - λ v ω 3 - - λ 4 ω 4 + v 4 λ 2 4

das heißt, alle Interaktionsterme zwischen EIN EIN und ω ω verschwinden damit ω ω wird zu einem automatisch wechselwirkenden Feld mit unendlicher Masse, das vom Rest der Theorie entkoppelt ist und daher keine Rolle spielt. So gewinnen wir das massive Vektorfeld zurück, mit dem wir begonnen haben.

L = - 1 4 F. 2 + a 2 2 EIN 2 L. = - - 1 4 F. 2 + ein 2 2 EIN 2

Beachten Sie, dass in einer nicht-abelschen Eichentheorie nichtlineare Begriffe wie z g EIN 2 EIN G EIN 2 EIN , g 2 EIN 4 G 2 EIN 4 , die uns daran hindern, das Limit zu überschreiten G 0 G 0 .

Vielen Dank für die umfassende Antwort. Es scheint, dass Sie meiner Aussage zustimmen, dass die Identität der Gemeinde eine Folge der gegenwärtigen Erhaltung oder gleichwertig ist U. ( 1 ) U. ( 1 ) globale Symmetrie, aber nicht wirklich von der gemessenen (oder lokalen) Symmetrie abgeleitet. Und wie ist der Stuekelberg-Mechanismus für meine Frage relevant?
Außerdem scheint die Ward-Identität etwas mehr zu sagen als "Photonenpolarisationen parallel zur Photonenausbreitung tragen nicht zu Streuamplituden bei", da bei Streuamplituden der betrachtete Impuls des externen Photons auf der Schale liegt, während die Ward-Identität sogar für die Off-Shell gilt k μ k μ
@ JiaYiyang Ich stimme nicht zu. Die Invarianz des Messgeräts ist eine Voraussetzung. Beachten Sie, dass es in einer Theorie mit einem komplexen Skalarfeld mit einer quartischen Selbstinteraktion eine globale gibt U. ( 1 ) U. ( 1 ) Symmetrie, aber es gibt keine Ward-Identität, die im Geiste der QED-Identität ähnlich ist (es gibt eine Ward-Identität, die besagt, dass die Anzahl der Partikel abzüglich der Anzahl der Antiteilchen erhalten bleibt). Das gleiche Problem tritt auf, wenn man ersetzt F. μ ν F. μ ν mit μ EIN ν - b ν EIN μ μ EIN ν - - b ν EIN μ mit b 1 b 1 in QED. Man hat noch j = 0 j = 0 , aber keine QED-ähnliche Ward-Identität.
Ich denke, dass der Fall von QED plus einem Massenterm für das Photon etwas ganz Besonderes ist, da es eine versteckte Eichensymmetrie gibt. In meiner Darstellung ist diese verborgene Eichsymmetrie durch die Transformation gegeben B B + ( 1 / g ) α B. B. + ( 1 /. G ) α , χ χ + α χ χ + α . Ich denke, das beantwortet auch Ihre letzte Frage.
Übrigens, ich freue mich über Ihre Fragen. Sie sind gut. @ JiaYiyang

Lassen Sie mich versuchen, meine eigene Frage zu beantworten, nachdem ich einige Zeit mit dem Lesen von L. Brown's "Quantenfeldtheorie" verbracht habe, aber ich werde mich nicht an seine Notationen halten.

Lassen Sie mich ein wenig auf die Terminologie eingehen, die ich verwenden werde: "Generalized Ward Identity ( GWI )" bezieht sich auf ( l - k ) μ Γ μ ( k , l ) = i S. - 1 ( k ' ) - i S. - 1 ( l ) ( l - - k ) μ Γ μ ( k , l ) = ich S. - - 1 ( k ' ) - - ich S. - - 1 ( l ) , wo Γ μ ( k , l ) Γ μ ( k , l ) ist eine Elektronen-Elektronen-Photonen-Scheitelpunktfunktion, S. S. ist ein (vollständiger) Elektronen-Elektronen-Propagator. Darauf werde ich später noch näher eingehen. "Ward Identity ( WI )" bezieht sich auf den Sonderfall, wenn man lässt l k l k in GWI; "Ward-Takahashi-Identität ( WTI )" bezieht sich auf k μ M. μ ( k ) = 0 k μ M. μ ( k ) = 0 .

Ich sollte gestehen, als ich diese Frage stellte und als ich die Worte "... behaupten, dass die Identität der Gemeinde eine Folge der Eichinvarianz der Theorie ist" formulierte, wusste ich nicht, auf welche der drei Identitäten sie sich bezogen, aber jetzt Zumindest kann ich sagen, dass GWI wirklich eine Folge der Eichinvarianz ist, nicht der globalen Phasensymmetrie. Kurz gesagt, wenn Γ μ Γ μ in GWI wird als unpassender Scheitelpunkt (dh reduzierbarer Scheitelpunkt mit 1 Teilchen) genommen, dann gilt GWI für Theorien, die die Stromerhaltung (oder die globale Phasensymmetrie) berücksichtigen. Für die Theorie mit einer Eichsymmetrie erhalten wir jedoch einen stärkeren GWI, dh der GWI gilt nicht nur für einen falschen Scheitelpunkt, sondern auch für den richtigen (dh einen irreduziblen 1-Teilchen-Scheitelpunkt).

GWI von Improper Vertex

Lassen Sie uns zuerst sehen, wie Sie den GWI für die aktuelle Konservierung erhalten, und hier werde ich im Grunde genommen von Weinberg Vol. I, Kapitel 10, kopieren ⟨T. { J. μ ( x ) Ψ n ( y ) Ψ ¯ m ( z ) } T. { J. μ ( x ) Ψ n ( y ) Ψ ¯ m ( z ) }} . Diagrammatisch ist dies die Summe aller Diagramme mit 1 externen Photonenpropagator und 2 externen Elektronenpropagatoren, jedoch mit einem bloßen externen Photonenpropagator, der entfernt wurde. Jetzt definiert Weinberg Γ μ ( k , l ) Γ μ ( k , l ) durch

d 4 x d 4 y d 4 z e - i p x e - Ich k y e i l z ⟨T. { J. μ ( x ) Ψ n ( y ) Ψ ¯ m ( z ) } - i q S. n n ' ( k ) Γ μ n ' m ' ( k , l ) S. m ' m ( l ) δ 4 ( p + k - l ) d 4 x d 4 y d 4 z e - - ich p x e - - ich k y e ich l z T. { J. μ ( x ) Ψ n ( y ) Ψ ¯ m ( z ) }} - - ich q S. n n ' ( k ) Γ n ' m ' μ ( k , l ) S. m ' m ( l ) δ 4 ( p + k - - l )

wo S. n m S. n m ist die Fourier-Transformation von T. { Ψ n ( y ) Ψ ¯ m ( z ) } T. { Ψ n ( y ) Ψ ¯ m ( z ) }} (und lassen Sie eine Delta-Funktion weg), so ist es der volle Elektronenpropagator. Jetzt können wir sehen Γ μ Γ μ ist die Scheitelpunktfunktion, nachdem 2 Vollelektronenpropagatoren und 1 bloßer Photonenpropagator abgestreift wurden, so dass sie entlang der Photonenlinie 1 Teilchen reduzierbar sind (dh immer noch die Photonenvakuumpolarisationskorrektur enthalten), daher nicht korrekt. Diagramme sind: Geben Sie hier die Bildbeschreibung ein

wobei eine gestrichelte Linie bedeutet, dass die Linie entfernt wurde, und Γ μ P. Γ P. μ bezeichnet den richtigen Scheitelpunkt, und wir können bekommen Γ μ P. Γ P. μ wenn wir den Photonenvakuum-Polarisationsteil weiter abstreifen können. Der Rest ergibt sich grundsätzlich aus der Berechnung x μ ⟨T. { J. μ ( x ) Ψ n ( y ) Ψ ¯ m ( z ) } x μ T. { J. μ ( x ) Ψ n ( y ) Ψ ¯ m ( z ) }} , bewirbt sich μ J. μ = 0 μ J. μ = 0 und dann eine Fourier-Transformation.

GWI von Proper Vertex

Jetzt werde ich für die Theorie mit lokaler Eichinvarianz behaupten, GWI gilt auch für den richtigen Scheitelpunkt Γ μ P. Γ P. μ . Die Idee ist zu isolieren Γ μ P. Γ P. μ von Γ μ Γ μ . Wie aus der zweiten Abbildung leicht ersichtlich ist, können wir zuerst den nackten Photonenpropagator wieder hinzufügen (bezeichnen wir ihn mit G μ ν 0 G 0 μ ν ) und entfernen Sie dann einen vollständigen Photonenpropagator G μ ν G μ ν , das ist,

Γ μ P. ( k , l ) = G. - 1 ( p ) μ    ν G ν ρ 0 ( p ) Γ ρ ( k , l ) , Γ P. μ ( k , l ) = G - - 1 ( p ) ν μ G 0 ν ρ ( p ) Γ ρ ( k , l ) ,
wo p = l - k p = l - - k .

Um die LHS von GWI nachzuahmen, haben wir

( l - k ) μ Γ μ P. ( k , l ) = p μ G - 1 ( p ) μ    ν G ν ρ 0 ( p ) Γ ρ ( k , l ) ( ) . ( l - - k ) μ Γ P. μ ( k , l ) = p μ G - - 1 ( p ) ν μ G 0 ν ρ ( p ) Γ ρ ( k , l ) ( ) .
Hier kommt nun die Eichinvarianz ins Spiel:

Aussage: Eichinvarianz p μ G - 1 ( p ) μ    ν G ν ρ 0 ( p ) = p ρ . Aussage: Eichinvarianz p μ G - - 1 ( p ) ν μ G 0 ν ρ ( p ) = p ρ .

Wenn die Aussage wahr ist, erhalten wir sofort aus der Gleichung ( ) ( ) Das

p μ Γ μ P. ( k , l ) = p ρ Γ ρ ( k , l ) , p μ Γ P. μ ( k , l ) = p ρ Γ ρ ( k , l ) ,
und da gilt GWI für Γ ρ ( k , l ) Γ ρ ( k , l ) , von hier können wir schließen, dass es auch gilt für Γ μ P. ( k , l ) Γ P. μ ( k , l ) .

Hier ist die Skizze des Beweises der obigen Aussage: Mit einem Messparameter ξ ξ können wir die Umkehrung des nackten Propagators als schreiben

G - 1 0 ( p ) μ ν = ( g μ ν p 2 - p μ p ν ) - 1 ξ p μ p ν . G 0 - - 1 ( p ) μ ν = ( G μ ν p 2 - - p μ p ν ) - - 1 ξ p μ p ν .
Aufgrund der Eichinvarianz unterscheidet sich der volle Propagator nur vom bloßen im Querteil, und der Längsteil bleibt derselbe, d. H.
G - 1 ( p ) μ ν = ( g μ ν p 2 - p μ p ν ) F. ( p 2 ) - 1 ξ p μ p ν . G - - 1 ( p ) μ ν = ( G μ ν p 2 - - p μ p ν ) F. ( p 2 ) - - 1 ξ p μ p ν .
Dieser Satz selbst beinhaltet einen weiteren nicht ganz so kurzen Beweis, und er ist in Browns Buch zu finden. Der entscheidende Punkt ist jedoch, dass globale Symmetrie nicht ausreicht, um zu beweisen, dass man lokale Symmetrie benötigt. Stecken Sie die allgemeinen Formen der beiden Propagatoren ein, man kann die Aussage leicht beweisen.

Dies steht im Gegensatz zu einer Theorie ohne Eichinvarianz (z. B. können Sie den Propagtor eines massiven Vektorfeldes durch Ersetzen erhalten 1 ξ m 2 1 ξ m 2 ), dort wird der volle Propagator auch den Längsteil so verändern, dass er wird

G - 1 ( p ) μ ν = ( g μ ν p 2 - p μ p ν ) F. ( p 2 ) - 1 ξ H. ( p 2 ) p μ p ν , G - - 1 ( p ) μ ν = ( G μ ν p 2 - - p μ p ν ) F. ( p 2 ) - - 1 ξ H. ( p 2 ) p μ p ν ,
Wenn Sie dann die Berechnung in der Anweisung durchführen, erhalten Sie so etwas wie p μ G - 1 ( p ) μ    ν G ν ρ 0 ( p ) = H. ( p 2 ) p ρ p μ G - - 1 ( p ) ν μ G 0 ν ρ ( p ) = H. ( p 2 ) p ρ (oder vielleicht 1 H. ( p 2 ) p ρ 1 H. ( p 2 ) p ρ kann mich nicht ganz erinnern). Dann wird für den richtigen Scheitelpunkt GWI auf geändert
p μ Γ μ P. ( k , l ) = H. ( p 2 ) [ i S. - 1 ( k ' ) - i S. - 1 ( l ) ] , p μ Γ P. μ ( k , l ) = H. ( p 2 ) [ ich S. - - 1 ( k ' ) - - ich S. - - 1 ( l ) ]] ,
was nicht allzu gut ist, zB kann man die schöne Renormierungsrelation nicht erhalten Z. 1 = Z. 2 Z. 1 = Z. 2 für den richtigen Scheitelpunkt. Dies bedeutet auch, dass wir in der Eichentheorie die (richtige) Vertex-Renormierung getrennt von der Vakuumpolarisation betrachten können, während beispielsweise in der massiven Vektortheorie die Vakuumpolarisation berücksichtigt werden muss.

PS : Brown liefert auch einen zweiten Beweis für den richtigen Scheitelpunkt-WI, indem er eine effektive Aktionstechnik verwendet, die in gewisser Weise "kürzer" ist. Es erfordert jedoch viel mehr vorläufiges Wissen über effektive Maßnahmen und ist auch nicht so praktisch, um die Rolle der Eichinvarianz und der aktuellen Erhaltung im GWI zu kontrastieren. Daher habe ich die Methode hier nicht übernommen.

@drake: zögern Sie nicht zu kommentieren.
Gute Antwort! Ich denke, wenn der Massenbegriff durch einen Higgs-ähnlichen Mechanismus erzeugt wird, gilt auch die Identität, wie NAKANISHIs Artikel (in Motls Antwort verlinkt) hervorhebt. Was denken Sie?
@drake: Ich kann den Artikel nicht vollständig verstehen, da ich kein spontanes Brechen der Symmetrie gelernt habe und der Artikel nicht vollständig in sich geschlossen ist. Aber ich denke ja, da NAKANISHI über den richtigen Scheitelpunkt sprach.
Entschuldigen Sie diese Frage, aber ich bin Ihnen dankbar, wenn Sie antworten. Weinberg nimmt an, dass Grün funktioniert
d 4 x d 4 y d 4 z e i p x + i k y - i l z | T. ^ ( J. ^ μ ( x ) Ψ ^ n ( y ) Ψ ^ ¯ m ( z ) ) | (1) (1) d 4 x d 4 y d 4 z e ich p x + ich k y - - ich l z | T. ^ ( J. ^ μ ( x ) Ψ ^ n ( y ) Ψ ^ ¯ m ( z ) ) |
enthält die Summe der Diagramme mit zwei elektronischen Vollpropagatoren und einer photonischen Außenleitung. Aber ich verstehe nicht EIN ^ μ EIN ^ μ unter dem Zeitbestellvorgang ( J. ^ μ J. ^ μ enthält es nicht) also verstehe ich nicht warum Diagramme ( 1 ) ( 1 ) externe photonische Linie enthalten. Kannst du das erklären?
@ JiaYiyang, ausgezeichnet! Das ist eine wirklich beeindruckende Antwort. Ich habe mich schon lange gefragt, wie das WI im Strom und das im richtigen Scheitelpunkt zusammenhängen. Ja, jetzt hat Ihre Antwort tatsächlich gezeigt, dass der richtige Scheitelpunkt WI von der Eichsymmetrie abhängt, nicht nur von der globalen Symmetrie!

Die Ward-Identität ergibt sich aus der Eichsymmetrie und es ist möglich, diese Dinge zu sehen, ohne irgendeinen Strom zu erwähnen. Die Ward-Identität sagt k μ M. μ ( k ) = 0 k μ M. μ ( k ) = 0 was wirklich sagt, dass die longitudinale Polarisation des Eichbosons, eine mit dem reinen Eichpolarisationsvektor proportional zum Impuls, ϵ μ k μ ϵ μ k μ , "entkoppelt" dh seine Wechselwirkungen (Streuamplituden) mit einer Ansammlung physikalischer Teilchen verschwinden.

Dieses Verschwinden impliziert eine Symmetrie - jetzt ja, k μ M. μ ( k ) k μ M. μ ( k ) kann auch als Korrelator einschließlich interpretiert werden μ J. μ μ J. μ , ein konservierter Strom - und diese Symmetrie ist eine Eichsymmetrie, da das Eichfeld möglicherweise nur einen Wert ungleich Null hat k μ k μ dh Abhängigkeit von der Raumzeit, wenn wir zulassen, dass der Symmetrieparameter von der Raumzeit abhängt.

Felder mit einem Extra m 2 EIN μ EIN μ m 2 EIN μ EIN μ usw. sind nicht mehr an einen konservierten Strom gekoppelt, da der Strom durch einen zusätzlichen Strom modifiziert wird m 2 EIN μ m 2 EIN μ - weil dies als Multiplikationsfaktor erscheint EIN μ EIN μ in einem Begriff, den Sie gerade hinzugefügt haben - was auch bedeutet, dass die Ward-Identität nicht gilt, wenn Sie die Symmetrie auf diese explizite Weise brechen (die Ward-Identität wird " kontrollierbarer " gebrochen, wenn Sie die Symmetrie spontan und nicht explizit brechen, weil der Lagrange vollständig ist hat immer noch die Eichsymmetrie, dh das Eichfeld ist an einen konservierten Strom gekoppelt).

Ich stimme Ihrem ersten Absatz voll und ganz zu, dh die Eichsymmetrie impliziert die Identität der Gemeinde. Aber ich verstehe Ihren zweiten Absatz nicht, dh die Identität der Gemeinde impliziert eine Eichsymmetrie. Ich wäre Ihnen dankbar, wenn Sie mehr ausarbeiten könnten. Ich stimme Ihrem dritten Absatz nicht zu. Wenn ich Sie richtig verstehe, meinen Sie m 2 EIN μ EIN μ + A. μ J. μ = A. μ ( m 2 EIN μ + J. μ ) m 2 EIN μ EIN μ + EIN μ J. μ = EIN μ ( m 2 EIN μ + J. μ ) . Sollte jedoch nicht m 2 EIN μ EIN μ m 2 EIN μ EIN μ eher als Teil des freien Lagrange als als des Stroms interpretiert werden?
Ich habe gerade eine Antwort geschrieben, die teilweise nicht mit Ihrer übereinstimmt. Es wäre schön, wenn Sie auf einen Fehler hinweisen könnten, den Sie in meiner Ableitung oder Argumentation sehen.